N210/220 Assignment 3: Health & Physical Assessment

Lakukan tugas rumah & ujian kamu dengan baik sekarang menggunakan Quizwiz!

While assessing a client's vascular system, the nurse finds that pulse strength is diminished or barely palpable. Which documentation is appropriate in this situation?

1+ A diminished or barely palpable pulse is documented as 1+. A normal and expected pulse strength is documented as 2+. A full, strong pulse is documented as 3+. A bounding pulse is documented as 4+.

While caring for a client with heat stroke, the nurse measured the temperature and noted it as 39ºC. What is this temperature in Fahrenheit?

102.2 F Celsius is converted to Fahrenheit by multiplying the Celsius reading by 9/5 and adding the product to 32. In this case, the calculation is: (9/5)(39) + 32 = 102.2.

What is the appropriate blood pressure of a 12-year-old client?

110/65 mm Hg A 12-year-old client typically has a blood pressure of 110/65 mm Hg. A 1-year-old client would typically have a blood pressure of 95/65 mm Hg. A 6-year-old client would typically have a blood pressure of 105/65 mm Hg. A 14- to 17-year-old client has a typical blood pressure of 119/75 mm Hg.

A client weighs 150 lb and is 5 feet 7 inches tall. What is this client's body mass index (BMI)? Record your answer using two decimal places.

23.53 Body mass index (BMI) can be calculated by dividing the client's weight in kilograms by the height in meters squared. Therefore, a client who weighs 150 lb (68 kg) and stands 5 feet 7 inches (1.7 m) tall will have a BMI of 23.53: 68/1.7 2 = 23.53.

What would be the respiratory rate in two-year-old child?

30 The normal range for the respiratory rate in a two-year-old kid (toddler) is between 25 and 32 breaths per minute. Twenty breaths per minute is the normal respiratory rate in adolescents and adults. The normal respiratory rate in newborns is 40. The normal respiratory rate in infants is 50 breaths per minute.

An older adult with chills arrived to hospital. The nurse assesses the client's vital signs and determined the client has a fever. What would be the client's rectal temperature?

38.5 degrees C (101.3 degrees F) In older adults the normal temperature range is 36° to 36.8°C orally (96.8-98.24 F) and 36.6° to 37.2°C rectally (97.88-98.96 F). In febrile conditions, the rectal temperature would be more than 37.5°C (99.5 F). A rectal temperature of 38.5°C (101.3 F) would indicate a fever.

A nurse is caring for a client who has been admitted with right-sided heart failure. The nurse notes that the client has dependent edema around the area of the feet and ankles. To characterize the severity of the edema, the nurse presses the medial malleolus area, noting an 8 mm depression after release. How should the edema be documented?

4+ Dependent edema around the area of feet and ankles often indicates right-sided heart failure or venous insufficiency. The nurse should assess for pitting edema by pressing firmly for several seconds, then releasing to assess for any depression left on the skin. The grading of 1+ to 4+ characterizes the severity of the edema. A grade of 4+ indicates an 8 mm depression. A grade of 1+ indicates a 2 mm depression. A grade of 2+ indicates a 4 mm depression. A grade of 3+ indicates a 6 mm depression.

A nursing student is recording the radial pulse rate in a client with dysrhythmias and documented a radial pulse of 80 beats per minute. The registered nurse reassesses the client and notices a pulse deficit of 15. What is the client's apical pulse?

95 Dysrhythmias are often associated with pulse deficits. A pulse deficit is the difference between the apical and radial pulse rates. Thus, when the radial pulse (80) and the pulse deficit (15) are added together, the apical pulse would be 95.

While assessing a client who experienced an accident, the nurse found that the client is unable to move eyeballs laterally. Which nerve damage led to this condition in the client?

Abducens nerve The abducens nerve is the VI cranial nerve, which helps in lateral movement of the eyeballs. Damage to this nerve limits lateral movement of the eyeball. Injury to the optic nerve causes changes in visual acuity. Injury to the facial nerve results in loss of facial expressions and loss of taste perception from the anterior one third of the tongue. Injury to the oculomotor nerve limits the extraocular movements and pupillary responses.

A client complains of chronically recurring ulcers on the lower leg. Upon assessment, a nurse finds the absence of hair growth on the legs and asks the client to consult the primary healthcare provider immediately. Which condition does the nurse suspect?

Circulatory insufficiency Chronically recurring ulcers in the lower legs or the feet accompanied by an absence of leg hair growth are serious signs of circulatory insufficiency. This finding requires a primary healthcare provider's intervention. The symptoms of phlebitis include localized redness, tenderness, and swelling over the vein sites of the calves. Clubbing is caused by insufficient oxygenation at the periphery. This can result from chronic emphysema and congenital heart disease. Occlusion is characterized by pain, pallor, pulselessness, paresthesias, and paralysis.

A nurse is evaluating the effectiveness of treatment for a client with excessive fluid volume. What clinical finding indicates that treatment has been successful?

Clear breath sounds Excess fluid can move into the lungs, causing crackles; clear breath sounds support that treatment was effective. Although it may make palpation more difficult, excess fluid will not diminish pedal pulses. A normal potassium level can be maintained independently of fluid excess correction. As the client excretes excess fluid, the urine specific gravity will increase, not decrease.

A client complains of pain in the ear. While examining the client, a nurse finds swelling in front of the left ear. Which lymph node does the nurse expect to be involved?

Pre-auricular The pre-auricular lymph node is located in front of the ear and in this situation would be edematous. The mastoid or posterior auricular lymph node is present behind the ear. The occipital lymph nodes are located in the back of the head, near the occipital bone of the skull. Submental lymph nodes are located below the chin.

After an eye assessment, the nurse finds that both of the client's eyes are not focusing on an object simultaneously and appear crossed. What could be the cause for this condition?

impairment of extraocular muscles Strabismus is a condition where the eyes appear crossed; this condition is caused by the impairment of the extraocular muscles. A loss of lens elasticity may lead to presbyopia, which causes impaired near vision. An obstruction of the aqueous humor outflow may lead to glaucoma. The progressive degeneration of the center of the retina indicates macular degeneration and leads to blurred central vision.

A nurse suspects that a client has interacted with poison ivy. Assessment findings reveal vesicles on the arms and legs. Which is the description of a vesicle?

lesion filled with serous fluid A vesicle is a small blisterlike elevation on the skin containing serous fluid. Vesicles are usually transparent. Common causes of vesicles include herpes, herpes zoster, and dermatitis associated with poison oak or ivy. A lesion filled with purulent drainage is known as a pustule, an erosion into the dermis is known as an excoriation or ulcer, and a solid mass of fibrous tissue is known as a papule.

The nurse is performing a skin assessment of a client. Which findings in the client may indicate a risk of skin cancer? Select all that apply. lesions lumps rashes bruising dryness

lesions lumps rashes Lesions on the skin that take a long time to heal may indicate skin cancer. Lumps and rashes on the skin are characteristics of skin cancer. Bruising may indicate a bleeding disorder or injury. Dryness of the skin may be due to excessive bathing and use of harsh soaps.

A client suffers hypoxia and a resultant increase in deoxygenated hemoglobin in the blood. What are the best sites to assess this condition? Select all that apply. lips sclera mouth sacrum nail beds shoulders

lips mouth nail beds Prolonged hypoxia resulting in increased amounts of deoxygenated blood causes cyanosis, which can be best evaluated in lips, mouth, nail beds, and skin (in extreme conditions). Sclera is the site of assessment for jaundice, while shoulders are assessed to confirm the condition of erythema.

A nurse is assessing a client with a history of marijuana use. Which long-term effects are associated with marijuana? Select all that apply. lung cancer emphysema heart disease laryngeal disorder stroke chronic nasal irritation

lung cancer emphysema heart disease Lung cancer, emphysema, and heart disease are outcomes that may occur due to marijuana use. Laryngeal disorders, stroke, and chronic nasal irritation are associated with the abuse of cocaine but are not associated with marijuana use.

While assessing a client, a nurse finds that the ratio of the anteroposterior diameter and transverse diameter of the chest is 1:1. What is indicated by this finding? Select all that apply. Client has/is... lordosis an older adult osteoporosis history of smoking chronic lung disease

older adult history of smoking chronic lung disease The 1:1 ratio of the anteroposterior diameter and transverse diameter of the chest indicates a barrel-shaped chest. This is a characteristic feature in an older adult who smokes and has chronic lung disease. In lordosis, there is an increase in lumbar curvature. Osteoporosis is a systemic skeletal condition in which there is a decreased bone mass and deterioration of bone tissue.

Which clients suffer from impaired near vision? Select all that apply. client with... myopia presbyopia hyperopia retinopathy macular degeneration

presbyopia hyperopia A loss of elasticity of the lens causes impaired near-vision in presbyopia. Light rays focusing behind the retina are the cause of impaired near vision in clients with hyperopia. Myopia is caused by a refractive error where the light rays focus in front of the retina. Retinopathy is a noninflammatory change in the retinal blood vessels. Macular degeneration is a blurring of central vision caused by progressive degeneration of the central retina.

How does the World Health Organization (WHO) define "health"?

A state of complete physical, mental, and social well-being The WHO defines health as a "state of complete physical, mental, and social well-being, not merely the absence of disease or infirmity." Pender explains that all people free of disease are not healthy. Pender, Murdaugh, and Parsons suggest that for many people, health is a condition of life rather than pathological state. Life conditions such as environment, diet, or lifestyle choices can have positive or negative effects on health long before an illness is evident. Pender, Murdaugh, and Parsons (2011) define health as the actualization of inherent and acquired human potential through goal-directed behavior, competent self-care, and satisfying relationships with others.

An older adult is found to have a thin white ring around the margin of the iris. What condition does this denote?

Arcus senilis In older adults, the iris becomes faded and a thin white ring (known as arcus senilis) appears around the margin of the iris. A cataract is a condition involving increased opacity of the lens that blocks light rays from entering the eye. The presence of redness indicates allergic or infectious conjunctivitis. Macular degeneration is marked by a blurring of central vision caused by progressive degeneration of the center of the retina.

The student nurse prepares a concept map while caring for a client recovering from surgery. What is the first step that the student nurse should take when preparing the concept map?

Arrange cues into clusters that form patterns A concept map is a visual representation of the connection between the client's many health problems. The first step is to arrange all the cues into clusters that form patterns. This helps the nurse identify specific nursing diagnoses for the client. During the assessment stage, the nurse assesses the client and gathers information. This step is performed before preparing the concept map. After placing all cues into clusters, the nurse begins to identify patterns reflecting the client's problem. The concept map helps the nurse obtain a holistic view of the client's needs. The next step is to identify specific diagnoses so that appropriate nursing interventions can be provided.

A client complains of difficulty breathing. The nurse auscultates wheezing in the anterior bilateral upper lobes. What could be the possible reason for this sound?

High velocity airflow through an obstructed airway Wheezing is a high-pitched sound that may be caused by a high velocity airflow through an obstructed or narrowed airway. Inflammation of the pleura may produce pleural friction rubs. Muscular spasms in larger airways or any new growth causing turbulence may produce rhonchi, which is a loud and low-pitched sound. Sudden reinflation of groups of alveoli may produce crackling sounds.

While inspecting the external eye structure of a client, a nurse finds bulging of the eyes. Which condition can be suspected in the client?

Hyperthyroidism Bulging eyes may indicate hyperthyroidism. Tumors are characterized by abnormal eye protrusions. Hypothyroidism can be revealed by the coarseness of the hair of the eyebrows and the failure of the eyebrows to extend beyond the temporal canthus. Crossed eyes or strabismus may result from neuromuscular injury or inherited abnormalities.

A client with a head injury underwent a physical examination. The nurse observes that the client's temperature assessments do not correspond with the client's condition. An injury to which part of the brain may be the reason for this condition?

Hypothalamus The hypothalamus controls the body temperature. Damage to the hypothalamus may cause abnormalities in the body temperature values during a physical assessment. The pons is responsible for maintaining level of consciousness. The medulla controls heart rate and breathing. The thalamus performs motor and sensory functions.

Which type of breathing pattern alteration is manifested with hypercarbia? Hypercardia = Hypercapnia

Hypoventilation Hypercarbia may occur during hypoventilation. The respiratory rate is abnormally low and the depth of ventilation is depressed in hypoventilation. In eupnea, the normal rate and depth of respiration is interrupted while singing. The rate of breathing is regular, but abnormally rapid in tachypnea. Respirations are abnormally deep, regular, and the rate is increased in Kussmaul's respirations.

Which statement made by the nurse indicates that the client interview is coming to a close?

I just have one more question for you The nurse should give the client a clue that the interview is drawing to a close. The nurse can do this by letting the client know that after one more question the interview will be over. The nurse sets the stage for the interview by ensuring that the client is comfortable and not in pain. The nurse begins the interview by stating that he or she would like to spend some time to understand the client's health concerns. The nurse informs the client at the beginning of the interview that the information shared by the client is confidential.

A registered nurse is teaching a nursing student about precautions to be taken for physical examination of a client. Which statements made by the nursing student indicate effective learning? Select all that apply. I should examine the client in noise-free areas I should use latex gloves during the physical examination I should perform a physical examination in a cool room I should leave a combative client alone during a physical examination I should wear eye shields while examining a client with excessive drainage

I should examine the client in noise-free areas I should wear eye shields while examining a client with excessive drainage Clients should be examined in noise-free areas to prevent interruptions. Wearing eye shields while examining a client with excessive drainage helps to reduce contamination. Latex gloves should be used with caution because they may cause allergy in clients who are allergic to latex. A physical examination should be performed in a warm room to minimize discomfort. Combative clients should never be left alone during physical examinations.

Which nursing intervention is employed to encourage the client to fully reveal the nature of their health problem?

The nurse leans forward attentively during the discussion The nurse leans forward attentively to show awareness, attention, and immediacy during discussion with the client. This encourages the client to fully reveal the nature of his or her health problem. The client may feel that the nurse is too busy to pay attention if the nurse takes down notes during the discussion. If the nurse hurries with the assessment, the client is likely to understand that the nurse is busy or not interested. The nurse should ask open-ended questions that encourage the client to give detailed information about his or her health. Closed-ended questions that can be answered with "yes" or "no" often bring the conversation to a close.

A client who does not understand English requires an interpreter. Which nursing student action may exacerbate health disparities?

The student talks only to the interpreter about the client A nurse should follow certain strategies while working with an interpreter for a client who does not understand English. The nurse should talk to the client about the client's condition and care and not to the interpreter. The interpreter may act as a client advocate and represent the client's needs to the nurse. The nurse should use a trained medical interpreter who has a health care background. The nurse should maintain eye contact with the client and obtain feedback to be certain that the client understands.

Which statement is true for collaborative problems in a client receiving healthcare?

They are identified by the nurse during the nursing diagnosis stage The nurse assesses the client to gather information to reach diagnostic conclusions. Collaborative problems are identified by the nurse during this process. If the client's health problem requires treatment by other disciplines such as medical or physical therapy, the client has a collaborative problem. A medical diagnosis is the identification of a disease condition. Problems that require treatment by the nurse are referred to as nursing diagnoses. A medical diagnosis is identified by the primary healthcare provider based on the results of diagnostic tests.

A nurse is caring for a client with diarrhea. The nurse anticipates a decrease in which clinical indicator?

Tissue turgor Skin elasticity will decrease because of a decrease in interstitial fluid. The pulse rate will increase to oxygenate the body's cells. Specific gravity will increase because of the greater concentration of waste particles in the decreased amount of urine. The temperature will increase, not decrease.

While assessing a client, the nurse finds inflammation of the skin at the bases of the client's nails. What might be the reason behind this condition?

Trauma Paronychia is an abnormality of the nail bed. The condition is marked by inflammation of the skin at the base of the nail; this condition may be caused by trauma or a local infection. Trichinosis is associated with red or brown linear streaks in the nail bed. Pulmonary diseases can cause changes in the angle between nail and nail base, which is a phenomenon known as clubbing. Koilonychia, a concave curvature of the nails, may occur as a result of iron-deficiency anemia.

While assessing a client who sustained a road traffic accident, a nurse notices that the client is unable to clench his teeth. Which cranial nerve might have been affected?

Trigeminal nerve The trigeminal nerve provides sensory innervation to the facial skin and motor innervation to the muscles of the jaw. A client with a damaged trigeminal nerve will be unable to clench his teeth. The facial nerve provides sensory and motor innervations for facial expressions. The trochlear nerve is involved in downward and inward eye movements. The abducens nerve helps in the eyeball's lateral movement.

Which Korotkoff sound represents systolic blood pressure in children?

first The first Korotkoff sound represents the systolic pressure in everyone. A blowing or swishing sound occurs in the second Korotkoff sound. The third Korotkoff sound involves crisper and more intense tapping sounds. The fifth Korotkoff sound is the point at which sound disappears and represents the diastolic pressure in adults and adolescents.

Nurses care for clients in a variety of age groups. In which age group is the occurrence of chronic illness the greatest?

Older adults The incidence of chronic illness increases in older adults because of the multiple stresses of aging. Younger individuals have greater physiologic reserves, and chronic illnesses are not common.

The nurse cares for an unconscious client who underwent head surgery. Which site would be best used to monitor body temperature?

Rectal Although the oral route is the most common route for monitoring body temperature, clients who are unconscious should have their temperatures monitored rectally. Skin temperature may be impaired due to diaphoresis; this measurement may not reliable. The axilla temperature may underestimate the core temperature.

Which physical assessment of the skin indicates that a client is addicted to phencyclidine?

Red and dry skin Phencyclidine (PCP): drug used for its mind-altering effects. PCP may cause hallucinations, distorted perceptions of sounds, and violent behavior. Red and dry skin is associated with phencyclidine abuse. A client with alcohol abuse will have burns on the skin. Vasculitis is associated with cocaine abuse. Diaphoresis is associated with chronic abuse of sedative hypnotics.

Which activity by the community nurse can be considered an illness prevention strategy?

Arranging an immunization program for chicken pox An illness prevention program protects people from actual or potential threats to health. A chickenpox immunization program is an illness prevention program. It motivates the community to prevent a decline in health or functional levels. A health promotion program encourages the client to maintain the present levels of health. The nurse promotes the health of the client by encouraging the client to exercise daily. Wellness education teaches people how to care for themselves in a healthy manner. The nurse provides wellness education by teaching about stress management. The nurse promotes the health of the client by teaching the client to maintain a nutritious diet.

A nurse is caring for a client admitted with cardiovascular disease. During the assessment of the client's lower extremities, the nurse notes that the client has thin, shiny skin; decreased hair growth; and thickened toenails. What might this indicate?

Arterial insufficiency Clients experiencing arterial insufficiency present with extremities that become pale when elevated and dusky red when lowered. Lower extremities may also be cool to touch, pulses may be absent or mild, and skin may be shiny and thin with decreased hair growth and thickened nails. Clients with venous insufficiency often have normal-colored extremities, normal temperature, normal pulses, marked edema, and brown pigmentation around the ankles. Phlebitis is an inflammation of a vein that occurs most often after trauma to the vessel wall, infection, and immobilization. Lymphedema is swelling in one or more extremities that is a direct result of impaired flow of the lymphatic system.

A client presents with a shiny appearance of abdominal skin. The skin also has a taut appearance. Which condition may the client have?

Ascites Symptoms of ascites include a shiny and taut appearance of the abdominal skin. Cyanosis occurs when there is a bluish discoloration of the skin. Accidental injury and different types of bleeding disorders are characterized by bruises or needle marks on the skin.

The nurse is assessing a client who arrived at the healthcare facility for an appointment. Which action by the nurse will be beneficial during the interview?

Asking about the client's current concerns The nurse should begin the interview by gathering information about the client's current concerns to encourage the client to express his or her chief problems. The nurse then sets an agenda for the interview. However, the nurse must remember that the best interview focuses on the client and not the nurse's agenda. The nurse must ask open-ended questions that allow the client to describe his or her concerns more clearly. The nurse should ask the client to describe his or her healthcare expectations to help the client understand that the nurse is genuinely interested in the client's health.

The nurse is preparing to teach a client about self-injection of insulin. Which action by the nurse will increase the effectiveness of the teaching session?

Assess the client's barriers to learning self-injection techniques Before a teaching plan can be developed, the factors that interfere with learning must be identified. Although family members can be helpful, client involvement in care is most important for promoting independence and self-esteem. Assessment comes before intervention; written instructions may not be the most appropriate teaching modality. The client may never accept the change but must learn to manage care; this may be an unrealistic expectation.

Which clinical condition will result in changes in the integrity of the arterial walls and small blood vessels?

Atherosclerosis In atherosclerosis, there may be changes in the integrity of the walls of the arteries and smaller blood vessels. Direct manipulation of vessels or localized edema that impairs blood flow will lead to a contusion. Blood clotting that causes mechanical obstruction to blood flow indicates thrombosis. The tourniquet effect may be caused by the application of constricting devices, which may lead to impaired blood flow to areas below the site of constriction.

Which physical assessment technique involves listening to the sounds of the body?

Auscultation Auscultation involves listening to the sounds of the body. Palpation involves using the sense of touch to assess and collect data. An inspection involves the nurse carefully looking to collect data. Percussion involves tapping the skin with the fingertips to vibrate underlying tissues and organs.

While performing a physical assessment of a client, a nurse notices patchy areas with loss of pigmentation on the skin, hands, and arms. What is the probable etiology for this condition?

Autoimmune disease Patchy areas with loss of pigmentation on skin, hands, and arms are due to vitiligo, which is caused by an autoimmune or congenital disease. Anemia results in pallor due to a reduced amount of oxyhemoglobin. A tan-brown color of the skin is noticed in pregnancy due to an increased amount of melanin. Lung disease or heart failure can cause cyanosis due to an increased amount of deoxygenated hemoglobin.

A nurse is teaching a parenting class. What should the nurse suggest about managing the behavior of a young school-age child?

Be consistent about established rules Because of a short attention span and distractibility, consistent limit setting is essential toward providing an environment that promotes concentration, prevents confusion, and minimizes conflicts. Questions should be answered, but the answers should not be judgmental. A list of expectations may be overwhelming at this age. Parents need to assist children with routine tasks; children this age may not be concerned with time frames.

The nurse noticed the breathing rate as regular and slow while assessing a client for respiration. What could be the condition of the client?

Bradypnea In bradypnea the breathing rate is regular, but it is abnormally slow. Respirations cease for several seconds in apnea. The rate of breathing is regular, but abnormally rapid in tachypnea. In hyperpnea, the respirations are labored, the depth is increased, and the rate is increased.

Which term refers to a blowing sound created by turbulence caused by narrowing of arteries while assessing for carotid pulse?

Bruit A bruit is an audible vascular blowing sound associated with turbulent blood flow through a carotid artery. Ectropion is a condition in which the eyelid is turned outwards away from the eyeball. Entropion is a malposition resulting in an inversion of the eyelid margin. Borborygmi are rumbling or gurgling noises made by the movement of fluid and gas in the intestines

Which factor can elevate the oxygen saturation during an assessment?

Carbon monoxide Carbon monoxide artificially elevates the oxygen saturation during assessment. Nail polishes interfere with the ability of the oximeter. Intravascular dyes will artificially lower the oxygen saturation. Skin pigmentation will overestimate the saturation.

A client complains of sudden muscle weakness during times of anger or laughter that may occur at any time during the day. Which condition should be suspected in this client?

Cataplexy Cataplexy is a condition in which muscle weakness occurs suddenly during times of intense emotion, such as anger, sadness, or laughter. A cataplexic attack may occur at any time during the day. Insomnia is a condition in which a person has chronic difficulty falling asleep. A person with narcolepsy also experiences frequent awakenings from sleep, short periods of sleep or nonrestorative sleep, or some combination thereof. Narcolepsy is a dysfunction of the mechanisms that regulate sleeping and waking states. Sleep apnea is a disorder characterized by a lack of airflow through the nose and mouth for periods of 10 seconds or longer during sleep.

Which actions by the nurse help set the stage for a client-centered interview during the first visit after admission to the healthcare facility? Select all that apply. Close the door after entering the room Greet client using his or her last name Open the curtains to allow plenty of light in the room Introduce oneself with a smile and explain the reason for the visit Obtain an authorization from the client after the interview

Close the door after entering the room Greet client using his or her last name Introduce oneself with a smile and explain the reason for the visit The nurse should maintain the client's privacy by closing the door after entering the room. The nurse should maintain the dignity of the client by greeting the client using his or her last name. Smiling is a positive sign of warmth and immediacy when first establishing the nurse-client relationship. The nurse should explain his or her role in the providing care for the client. The nurse should ensure the room is adequately lit, comfortable, and soothing for the client. The nurse need not open the curtains to allow plenty of light in the room. The Health Insurance Portability and Accountability Act (HIPAA) requires the nurse to obtain an authorization from the client before collecting personal health data.

A client who underwent a physical examination reports itching after 2 days. Which condition should the nurse suspect?

Contact dermatitis A client who is allergic to latex may experience an allergy after a physical examination with latex gloves. Itching is one of the clinical signs of latex allergy. Contact dermatitis is a delayed immune response that occurs 12 to 48 hours after exposure. Eczema is a skin condition that can be worsened with excessive drying. Hypersensitivity is an immediate allergic reaction that occurs due to chemicals that are used to make gloves. Anaphylactic shock is also an immediate allergic reaction that occurs due to natural rubber latex.

While assessing an immobilized client, the nurse notes that the client has shortened muscles over a joint, preventing full extension. What is this condition known as?

Contracture Immobilized clients are at high risk for the development of contractures. Contractures are characterized by permanent shortening of the muscle covering a joint. Osteoarthritis is a disease process of the weight-bearing joints caused by wear and tear. Osteoporosis is a metabolic disease process in which the bones lose calcium. Muscle atrophy is a wasting and/or decrease in the strength and size of muscles because of a lack of physical activity or a neurologic or musculoskeletal disorder.

A nurse assesses the lungs of a client and auscultates soft, crackling, bubbling breath sounds that are more obvious on inspiration. This assessment should be documented as what?

Crackles Crackles are abnormal breath sounds described as soft, crackling, bubbling sounds produced by air moving across fluid in the alveoli. Vesicular breath sounds are normal. They are quiet, soft, and inspiration sounds that are short and almost silent on expiration. They are heard over the lung periphery. Bronchial breath sounds are normal and consist of a full inspiration and expiratory phase with the expiratory phase being louder. They are heard over the trachea and large bronchi of the lungs. Rhonchi are abnormal breath sounds heard over the large airways of the lungs. They consist of a low pitch and are caused by the movement of secretions in the larger airways; they usually clear with coughing.

A client who experienced extensive burns is receiving intravenous fluids to replace fluid loss. The nurse should monitor for which initial sign of fluid overload?

Crackles in the lungs Crackles, or rales, in the lungs are an early sign of pulmonary congestion and edema caused by fluid overload. Clients with fluid overload will usually demonstrate an increased heart rate and increased blood pressure. A decreased heart rate and decreased blood pressure and cyanosis in a client with fluid overload would be very late and fatal signs.

The nurse asks the client to shrug the shoulders and to turn the head against passive resistance. Which cranial nerve is involved in this action?

Cranial nerve XI Cranial nerve XI (the spinal accessory nerve) is the motor nerve that coordinates the movement of head and shoulders. Cranial nerve II (optic nerve) is a sensory nerve for visual acuity. Cranial nerve VI (abducens nerve) is a motor nerve that coordinates the lateral movement of eyeballs. Cranial nerve VII or (auditory nerve) is a sensory nerve which coordinates the hearing sense.

While assessing a neonate's temperature, the nurse observes a drop in the body temperature. What is the most appropriate reason for this temperature drop?

Decreased nonshivering thermogenesis Neonates are susceptible to heat loss or cold stress. Nonshivering thermogenesis is a natural mechanism of heat production that occurs to minimize heat loss in a neonate. This mechanism's failure may lead to a drop in body temperature. The basal metabolic rate (BMR) accounts for heat production; an increased BMR may raise the body temperature. Shivering is an involuntary movement that produces heat, which may not be seen in neonates. Voluntary movements cause increases in body temperature.

A client's breath has a sweet, fruity odor. Which condition is likely affecting this client?

Diabetic acidosis A client with diabetic acidosis has a sweet, fruity odor to the breath. Gum disease is marked by halitosis. A stale urine smell indicates uremic acidosis. An infection inside a cast is accompanied by a musty odor of the casted body part.

While assessing the eyes of a client, a healthcare provider notices there is an obstruction to the outflow of aqueous humor. Which additional finding might be noted to support a diagnosis of glaucoma?

Elevated intraocular pressure In glaucoma, there is an obstruction of the outflow of aqueous humor due to an intraocular structural damage, which may result from elevated intraocular pressure. Blurred central vision is seen in macular degeneration. Increased opacity of the lens may be seen in cataracts. Retinopathy may result from the changes in retinal blood vessels.

An 85-year-old client has just been admitted to a nursing home. When designing a plan of care for this older adult, the nurse recalls what expected sensory losses associated with aging? Select all that apply. Difficulty in swallowing Diminished sensation of pain Heightened response to stimuli Impaired hearing of high frequency sounds Increased ability to tolerate environmental heat

Diminished sensation of pain Impaired hearing of high frequency sounds Because of aging of the nervous system, an older adult has a diminished sensation of pain and may be unaware of a serious illness, thermal extremes, or excessive pressure. As people age they experience atrophy of the organ of Corti and cochlear neurons, loss of the sensory hair cells, and degeneration of the stria vascularis, which affects an older person's ability to perceive high frequency sounds. An interference with swallowing is a motor loss, not a sensory loss, and it is not an expected response to aging. There is a decreased, not heightened, response to stimuli in older adults. There is a decreased, not increased, ability to physiologically adjust to extremes in environmental temperature.

What clinical finding does a nurse anticipate when admitting a client with an extracellular fluid volume excess?

Distended jugular veins Because of fluid overload in the intravascular space, the neck veins become visibly distended. Rapid, thready pulse and elevated hematocrit level occur with a fluid deficit. If sodium causes fluid retention, its concentration is unchanged; if fluid is retained independently of sodium, its concentration is decreased.

A client presents with bilateral leg pain and cramping in the lower extremities. The client has a history of cardiovascular disease, diabetes, and varicose veins. To guide the assessment of the pain and cramping, the nurse should include which question when completing the initial assessment?

Does walking for long periods of time increase your pain? Clients with a medical history of heart disease, hypertension, phlebitis, diabetes, or varicose veins often develop vascular-related complications. The nurse should recognize that the relationship of symptoms to exercise will clarify whether the presenting problem is vascular or musculoskeletal. Pain caused by a vascular condition tends to increase with activity. Musculoskeletal pain is not usually relieved when exercise ends. Low potassium levels can cause cramping in the lower extremities; however, given the client's health history, vascular insufficiency should be suspected. Previously healed broken bones do not cause cramping and pain.

Which site should be monitored for a pulse to assess the status of circulation to the foot? Select all that apply.

Dorsalis pedis artery Posterior tibial artery The dorsalis pedis pulse and posterior tibial pulse are sites of assessments of circulation to the foot. The carotid pulse, located along the medial edge of the sternocleidomastoid muscle in the neck, is an easily accessible site to assess physiologic shock or cardiac arrest. The femoral artery pulse and popliteal artery pulses are helpful in assessing the circulation to the lower leg.

When should the nurse observe the client to assess his or her level of functioning? Select all that apply. During meal time When talking about pain When preparing medication During assessment interview When administering insulin injections

During meal time When preparing medication When administering insulin injections An observation of the functional level of the client often occurs during a return demonstration. The nurse may also observe the client while eating to determine if the client is able to eat without assistance. The nurse teaches the client how to prepare medications and asks for a return demonstration to assess the client's understanding. The nurse also observes the client administering insulin injections to ensure that the client is able to perform it properly. Observation of functional level differs from the observation during a physical examination. The nurse closely observes the client during the physical assessment when the client talks about pain. During the assessment interview, the nurse observes the client's facial expressions and eye contact to form accurate conclusions about the client's condition. The nurse does not assess the client's functional abilities during the subjective assessment.

The nurse is assessing a young client who presents with recurrent gastrointestinal disorders. On further assessment, the nurse learns that the client is experiencing job-related pressure. What is the most important nursing intervention for this client?

Educate the client on managing stress The client is experiencing job-related pressure, so the nurse should educate the client about managing stress as it is a lifestyle risk factor. Stress threatens both mental health and physical well-being. Stress is associated with illnesses such as heart disease, cancer, and gastrointestinal disorders. The nurse teaches the client to maintain a balanced diet as a primary preventive care to promote health. The nurse should instruct the client to have regular health checkups as a primary preventive measure. The nurse should ask the client to use sunscreen when working outdoors to avoid excess sun exposure and prevent skin cancer.

While performing a physical assessment of a female client, a nurse notices hair on the client's upper lip, chin, and cheeks. Which condition may result in this condition?

Endocrine disease Endocrine diseases such as hirsutism will result in excessive hair growth on the upper lip, chin, and cheeks. Aging and poor nutrition will result in decreased hair growth. Arterial insufficiency will result in decreased hair growth due to compromised blood supply.

The nurse is providing postoperative care to a client who had a submucosal resection (SMR) for a deviated septum. The nurse should monitor for what complication associated with this type of surgery?

Expectoration of blood After an SMR, hemorrhage from the area should be suspected if the client is swallowing frequently or expelling blood with saliva. A headache in the back of the head is not a complication of a submucosal resection. Crepitus is caused by leakage of air into tissue spaces; it is not an expected complication of SMR. The nerves and structures involved with speech are not within the operative area. However, the sound of the voice is altered temporarily by the presence of nasal packing and edema.

An elderly client is admitted to the healthcare facility following a stroke. What should the nurse do when the client's relative who arrived much later asks to see the client's health record?

Explain that medical health records are confidential The Health Insurance Portability and Accountability Act (HIPAA) sets the standards for the protection of the client's health information. The nurse must explain that medical health records are confidential. The healthcare team must be aware of the organization's policies for reviewing a client's medical record for assessment. The nurse need not confirm the client's relationship because the client's medical records are confidential and cannot be shared with anyone unless authorized by the client. The primary healthcare provider cannot authorize the nurse to show the medical records. The nurse cannot inform the nurse manager and show the medical record to persons not involved in direct client care. Healthcare providers share information with reasonable safeguards within the healthcare team for the purpose of providing client care.

During a survey, the community nurse meets a client who has not visited a gynecologist after the birth of her second child. The client says that her mother or sister never had annual gynecologic examinations. Which factor is influencing the client's health practice?

Family practices Family practices influence the client's perception of the seriousness of diseases. The client does not feel the need to seek preventive care measures because no family member practices preventive care. The client is not influenced by spiritual beliefs in this instance. An individual's spiritual beliefs and religious practices may restrict the use of certain forms of medical treatment. Emotional factors such as stress, depression, or fear may influence an individual's health practice; however, this client does not show signs of being affected by emotional factors. The client is said to be influenced by cultural background if he or she follows certain beliefs about the causes of illness and uses customary practices to restore health.

Which parts of the body assessed by the nurse would confirm a diagnosis of frostbite? Select all that apply. Axilla Fingers Ear lobes Forehead Upper thorax

Fingers Ear lobes Areas particularly susceptible to frostbite are the fingers, toes, and earlobes. These parts of the body should be assessed to determine frostbite. The axilla is generally used to assess the body temperature; this site is used to diagnose a fever. The forehead and upper thorax are assessed to detect diaphoresis.

While assessing a client with dehydration, the nurse notices diminished skin elasticity. Which portion of the hand is used to perform this assessment?

Fingertips The fingertips are used to palpate the skin for elasticity. The pads of the fingertips are used to palpate pulse amplitude. The ulnar surface of the hand is used to detect fremitus. The palmer surface of the fingertips is used to examine the thorax.

A nurse is caring for a client who is having diarrhea. To prevent an adverse outcome, the nurse should most closely monitor what client data or assessment finding?

Fluid and electrolyte balance Monitoring fluid and electrolyte balance is the most important nursing intervention because excess loss of fluid through the multiple loose bowel movements associated with diarrhea lead to alteration in fluid and electrolyte imbalance. Although skin may become excoriated with diarrhea, this is not a life-threatening condition and therefore not the nursing priority. Even though absorption of nutrients is decreased with diarrhea malnutrition, it is not a life-threatening condition and therefore not the priority nursing intervention. Fluid intake and output provides information about fluid balance only, without taking into consideration the loss of electrolytes that accompanies diarrhea and therefore is not the best choice.

A mother is worried about the sudden behavioral changes in her child. The child has suddenly developed a fear of certain people and places. The child's school performance is declining rapidly, and the child has developed poor relationships with his or her peers. After assessing the physical findings of the child, the nurse suspects child abuse. Which physical findings might have led the nurse to this suspicion?

Foreign bodies in the rectum, urethra, or vagina One of the physical findings that may be required to confirm child abuse is the presence of foreign bodies in the rectum, urethra, or vagina. Weight loss and sunken eyes may be a physical finding for older adult abuse. When the abuse is related to an intimate partner, the nurse may observe strangulation marks on the neck from rope burns or bruises. Staying isolated and not communicating with others are behavioral findings that may be related to older adult abuse.

What are the benefits of using standard formal nursing diagnostic statements? Select all that apply. Fosters development of nursing knowledge Allows nurses to communicate with the client Provides precise definition off the client's problem Distinguishes nurse's role from that of other care providers Enables primary healthcare provider to deliver effective health care

Fosters development of nursing knowledge Provides precise definition off the client's problem Distinguishes nurse's role from that of other care providers The use of standard formal nursing diagnostic statements fosters the development of nursing knowledge, which is important to be able to assess a client's specific risk for problems, identify them early, and take preventive action. Nursing diagnostic statements provide precise definitions of the client's problem. They give the nurses and other members of the healthcare team a common language for understanding the client's needs. Nursing is emphasized as an independent practice when the nurse formulates nursing diagnoses and individualized nursing care plans. This distinguishes the nurse's role from that of other care providers. Nursing diagnostic statements allow nurses to communicate what they do among themselves with other healthcare professionals and the public. A nursing diagnosis helps the nurse focus on the scope of nursing practice and to deliver effective healthcare.

After assessing the muscle functionality of a client, the nurse assigns a grade of F (fair) on the Lovett scale in the client. What is the muscle functionality of the client?

Full range of motion with gravity In the Lovett scale, grade F (fair) is given to clients who exhibit a full range of motion with gravity. Full range of motion in passive motion is assigned a P (poor) score. When a client exhibits full range of motion against gravity with full resistance, the client is given an N (normal) score. When a client exhibits full range of motion against gravity with marginal resistance, the client is given a score of G (good).

A client has a history of a persistent cough, hemoptysis, unexplained weight loss, fatigue, night sweats, and fever. Which risk should be assessed?

HIV infection A client with a history of persistent cough, hemoptysis, unexplained weight loss, fatigue, night sweats, or fever may have a human immunodeficiency virus (HIV) infection or tuberculosis. Lung cancer and cerebrovascular disease are risks to be assessed in the client with a history of tobacco or marijuana use. Cardiopulmonary alterations may be present in a client with a persistent cough (productive or nonproductive), sputum streaked with blood, or voice changes.

While assessing a client, the nurse finds bluish coloration of the skin. The nurse finds that this discoloration is due to cyanosis. Which condition may be suspected?

Heart disease A bluish discoloration of the skin indicates cyanosis. This condition may be caused by increased amounts of deoxygenated hemoglobin, which may lead in heart disease or lung disease. In clients with anemia, the skin has a pallor due to a reduced amount of oxyhemoglobin. In clients with liver disease, the skin appears yellow or orange due to increased deposits of bilirubin. In autoimmune diseases, the skin will lose its pigmentation.

An older adult with a history of diabetes reports giddiness, excessive thirst, and nausea. During an assessment, the nurse notices the client's body temperature as 105° F. Which condition does the nurse suspect in the client?

Heat stroke Older adults are more at a risk of heat stroke. Symptoms of heat stroke include giddiness, excessive thirst, nausea, and increased body temperature. Heat exhaustion is indicated by a fluid volume deficient. Heat exhaustion occurs when profuse diaphoresis results in excess water and electrolyte loss. Accidental hypothermia usually develops gradually and goes unnoticed for several hours. When the skin temperature drops below 95° F, the client suffers from uncontrolled shivering, memory loss, depression, and poor judgment. Malignant hyperthermia is an adverse effect of inhalational anesthesia that is indicated by a sudden rise in body temperature in intraoperative or postoperative clients.

The nurse tells a client undergoing diuretic therapy to avoid working in the garden on hot summer days. What condition is the nurse trying to prevent in this client?

Heatstroke Clients undergoing diuretic therapy are at risk of heatstroke when exposed to temperatures higher than 40° C. Frostbite occurs when the body is exposed to ice-cold temperatures. Hypothermia is a condition in which the skin temperature drops below 36° C. Hyperthermia occurs when the body is exposed to temperatures higher than 38.5° C.

A registered nurse is teaching a nursing student about when a client with high blood pressure should follow up with the primary healthcare provider. Which statement made by the nursing student indicates effective learning?

I will advise a client with a blood pressure of 130/80 mm Hg to follow up in a year A client with prehypertension tends to have a blood pressure (BP) between 120/80 and 139/89 mm Hg. These clients should be rechecked in a year. Clients with BP less than 120/80 mm Hg are considered normal. These clients should be rechecked in two years. Clients with stage 1 hypertension have a BP between 140/90 and 159/99 mm Hg. These clients should be rechecked in two months to confirm stage 1 hypertension. Clients with stage 2 hypertension have a BP greater than 160/100 mm Hg. These clients should be rechecked in one month. If a client's BP is greater than 180/110 mm Hg, then he or she should be treated immediately or within 1 week.

A nurse is teaching a client about proper hair hygiene and how to protect his or her hair from lice. Which statement made by the client indicates ineffective learning?

I will soak the comb used to remove lice for 15 minutes in cold water Any comb or brush that has been used to remove lice should be soaked for 15 minutes in very hot ammonia water or for 10 minutes in boiling water. A metal nit comb should be used to remove detectable nits after shampooing the hair to prevent further lice propagation. The hair should be thoroughly shampooed with pediculicide in cold water at a basin or sink to avoid contact of the medication with other body parts. Sealing items that are not washable in plastic bags should be done to prevent transmission of the lice.

A registered nurse teaches a nursing student about routines followed during a physical examination to help ensure that important findings are not missed. Which statement by the nursing student indicates ineffective learning?

I'll perform painful procedures at the beginning of the examination Any painful procedures should be performed at the end of the examination. The two sides of the body should be compared for symmetry, because some asymmetries are abnormal. Recording quick notes during the examination will help prevent delays during the examination. More extensive notes may be completed at the end of the examination. Assessments should be recorded in specific terms in the electronic or paper record. This standard form allows information to be recorded in the same sequence in which it is gathered.

A registered nurse is teaching a nursing student how to assess for edema. Which statement made by the student indicates the need for further education?

If the pressure on an edematous site leaves an indentation of 2 mm, a score of 2+ is given The depth of indentation left after applying pressure to an edematous site determines the degree of edema. A 1+ score is given if the depth of indentation is 2 mm. A 2+ is the score given if the depth of edema indentation is 4 mm. An accumulation of edematous fluid will result in the separation of skin and underlying vasculature. Edema is classified as pitting if the application of pressure on the edematous site will leave an indentation for some time. Edema results from a direct trauma to the tissue or by impaired venous return.

A registered nurse is teaching a nursing student about skin assessment. Which statement made by the nursing student indicates the need for further teaching?

In the absence of sunlight, skin assessments are performed best with other sources of light instead of fluorescent light Though skin assessments are best conducted in daylight, in the absence of sunlight, they are best performed in fluorescent lighting. Skin exposure during skin assessments in cool room temperature can result in cyanosis. Skin exposure during skin assessments made in warm room temperature can result in vasodilatation.

The nurse teaching a health awareness class identifies which situation as being the highest risk factor for the development of a deep vein thrombosis (DVT)?

Inactivity A DVT, or thrombus, may form as a result of venous stasis. It may lodge in a vein and can cause venous occlusion. Inactivity is a major cause of venous stasis leading to DVT. Pregnancy and tight clothing are also risk factors for DVT secondary to inactivity. Aerobic exercise is not a risk factor for DVT.

A nurse is preparing a community health program for senior citizens. The nurse teaches the group that what physical findings are typical in older adults?

Increased blood pressure and decreased cardiac output With aging, narrowing of the arteries causes some increase in the systolic and diastolic blood pressures. Decreases occur in diastolic pressure, diastolic filling, and beta-adrenergic stimulation; increases occur in arterial pressure, systolic pressure, wave velocity, and left ventricular end diastolic pressure. Decreased cardiac output and cardiac reserve decrease the older adult's response to stress. Changes in libido may occur. Testosterone appears to influence the frequency of nocturnal erections; however, low testosterone levels do not affect erections produced by erotic stimuli. There is a loss of skin elasticity. By the age of 60, gastric secretions decrease 70% to 80% of those of the average adult. A decrease in pepsin may hinder protein digestion. There may be a decrease in subcutaneous fat and decreasing body warmth. Some swallowing difficulties occur because older people are susceptible to fluid loss and electrolyte imbalance. This results from decreased thirst sensation, difficulty swallowing, chronic disease, reduced kidney function, diminished cognition, or adverse drug reactions.

The nurse recognizes that a common conflict experienced by older adults is the conflict between what?

Independence and dependence A common conflict confronting older adults is between the desire to be taken care of by others and the desire to be in charge of their own destiny. The conflict between the young and old age may occur but is not common. The conflict between the retirement and working may occur but is not common. The conflict between wishing to die and wishing to live may occur but is not common.

For which client(s) should the nurse consider family members as the primary source of information? Select all that apply. Older adult Infant or child During traumatic emergency When critically ill, disoriented In an outpatient

Infant or child During traumatic emergency When critically ill, disoriented The nurse interviews the parents who care for the infant or child. Thus, the parents become the primary source of information. A client who is brought to the emergency department after a trauma may not be in a position to explain the circumstances that led to the visit. In this case, the family or significant others who accompany the client become the primary source of information. The family becomes the primary source of information when the client is critically ill, disoriented, and unable to answer questions. Generally, the client is the primary source of information. The older adult who is conscious, alert, and able to answer the nurse's questions is the primary source of information. The client who visits the outpatient department is capable of providing accurate answers to the nurse's questions. This client is the primary source of information during assessment.

A nurse is caring for a client who underwent cardiac catheterization. The client's skin was found to be blanched, and there was formation of edema of 15.2 cm (1-6 inches) at the site of catheterization. Upon further assessment, the skin was found to be cool, and the client complains of tenderness. Which condition does the nurse expect?

Infiltration The client with blanched skin, edema of 15.2 cm, cool temperature, and pain at the site of catheterization has symptoms of grade 2 infiltration. Phlebitis is an inflammation of the inner layer of the vein. The findings for this include redness, tenderness, pain, and warmth along the course of the vein starting at the access site. If there is infection, there will be findings that include redness, heat, swelling at catheter-skin entry point, and possible purulent drainage. Circulatory overload can occur if intravenous solutions are infused too rapidly or in great amounts.

Which error will result in false high diastolic readings while measuring a client's blood pressure during a physical examination?

Inflating the cuff too slowly Inflating or deflating the cuff too slowly will yield false high diastolic readings. Wrapping the cuff too loosely will result in false high systolic and diastolic values. Applying the stethoscope too firmly will result in false low diastolic readings. Repeating the assessment too quickly will result in false high systolic readings.

A client has a fever spike that is combined with normal temperature levels. The client's body temperature returns to a normal body temperature at least once a day. Which type of fever can be assessed in the client?

Intermittent An intermittent fever is characterized by fever spikes interspersed with normal temperatures. In this type of fever, the body temperature returns to normal at least once in 24 hours. In the case of sustained fever, there is a constant body temperature greater than 38ºC. In relapsing fever, there is an occurrence of periods of febrile episodes with acceptable temperature values. In remittent fever, the body temperature increases and decreases without returning to normal body temperature levels.

A client shows an increase in rate respirations that are abnormally deep and regular. What condition would the nurse expect?

Kussmaul's respiration Kussmaul's respiration is an alteration in the breathing process that is characterized by an increased and abnormal deep and regular rate of respiration. A client suffering from hypoventilation would have an abnormally low respiratory rate and the depth of ventilation is depressed. In Biot's respiration, respirations are abnormally shallow for two to three breaths, followed by irregular periods of apnea. An irregular respiratory rate and depth characterized by alternating periods of apnea and hyperventilation would be observed in a client with Cheyne-Stokes respiration.

Which term refers to the exaggeration of the posterior curvature of the thoracic spine?

Kyphosis Kyphosis is an excessive outward curvature of the spine that causes hunching of the back. Lordosis is the excessive inward curvature of the lumbar part of the spine. Scoliosis is the abnormal lateral curvature of the spine. Osteoporosis is characterized by a loss of bone mass and a deterioration of bone tissues.

A nurse is assessing an 89-year-old client with a history of severe congenital spinal deformity. Which condition would most likely describe the nurse's finding?

Kyphosis Kyphosis is an increase in the curvature of the thoracic spine and may result from a congenital abnormality. Lordosis, also known as swayback, is an increased lumbar curvature and may not be a congenital abnormality. Presbycusis is the loss of acuity for high-frequency tones and is not related to the spine. Osteoporosis is a condition in which the bones become brittle and fragile from the loss of tissue and bone mass.

Which landmark is correct for a nurse to use when auscultating the mitral valve?

Left fifth intercostal space, midclavicular line The correct landmark for auscultating the mitral valve (apical pulse) is found at the left fifth intercostal space (ICS) in the midclavicular line. Auscultation at the fifth ICS in the midaxillary line would yield breath sounds of the lateral lung field. Auscultation at the left second ICS at the sternal border is best to hear the pulmonic valve, and at the left fifth ICS at the sternal border for the tricuspid valve.

A pregnant woman in her second trimester arrived at the hospital for a general health checkup. The physician recommended a pelvic examination to the client. Which position is most suitable for assessing the client in this condition?

Lithotomy position Lithotomy position provides maximum exposure to the female genitalia and easy examination of the region. Therefore this position is recommended for examining pregnant women. Sims position is indicated for rectal and vaginal examinations. Supine position is recommended for examining anterior thorax, lungs, breasts, axilla, heart abdomen, extremities, and pulse. Dorsal recumbent position is mainly indicated to examine the abdomen because it promotes abdominal relaxation.

After performing an optical assessment on a client, a primary healthcare provider notices impaired near vision. Which other finding in this client confirms the diagnosis as presbyopia?

Loss of elasticity of the lens Presbyopia is defined as impaired near vision caused by a loss of elasticity of the lens. This condition is reported in middle-aged and older adults. Increased opacity of the lens is seen in cataracts. Elevated intraocular pressure is associated with glaucoma. Retinopathy causes noninflammatory eye changes.

A nurse is assessing an older adult during a regular checkup. Which findings during the assessment are normal? Select all that apply. Loss of turgor Urinary incontinence Decreased night vision Decreased mobility of ribs Increased sensitivity to odors

Loss of turgor Decreased night vision Decreased mobility of ribs In older adults, the skin loses its turgor or elasticity and there is fat loss in the extremities. Visual acuity declines with age; therefore, decreased night vision is a normal finding in older adults. Decreased mobility of the ribs is found in older adults due to calcification of the costal cartilage. Urinary incontinence is an abnormal finding in older adults. In older adults, diminished sensitivity to odor, not increased sensitivity, is often found.

Which finding is inferred from a grade 4 intensity of heart murmurs?

Loud murmur associated with thrill Grade 4 indicates loud murmurs with an associated thrill. A thrill is a fine vibration that is felt by palpation. A grade 5 intensity is characterized by an easily palpable thrill. A grade 2 intensity is characterized by quiet and clearly audible murmurs. A moderately loud murmur without a thrill is noted as grade 3.

The nurse is caring for an African American client with renal failure. The client states that the illness is a punishment for sins. Which cultural health belief does the client communicate?

Magicoreligious belief An African American client may have magicoreligious beliefs, which focuses on hexes or supernatural forces that cause illness. Such clients may believe that illness is a punishment for sins. The yin/yang belief system does not consider illness as a punishment. The biomedical belief system maintains that health and illness are related to physical and biochemical processes with disease being a breakdown of the processes. The belief of determinism focuses on outcomes that are externally preordained and cannot be changed.

The nurse is caring for a client with a family history of diabetes mellitus. The client has been following a diet regimen recommended by the dietician and walking for 45 minutes daily for the past eight months. How should the nurse document the client's stage based on the transtheoretical model of health behavior change?

Maintenance The client is in the maintenance stage of human behavior change. During this stage, the client has managed to incorporate the changes in to the lifestyle. This stage begins six months after the action has started and continues indefinitely. The action stage lasts for six months from the time the client has incorporated the changes in to the lifestyle. During the preparation stage, the client begins to realize that the advantages of the change outweigh the disadvantages. The client starts making small changes in preparation for major changes the following month. During the contemplation stage, the client is still considering whether to incorporate changes in the next six months.

While assessing a client, a nurse finds adventitious breath sounds. Upon further evaluation, the nurse finds loud, low-pitched, rumbling coarse sounds during inspiration. This sound can be clearly heard while the client is coughing. What could be the reason behind these sounds?

Muscular spasms in the larger airways Adventitious breathing sounds ( rhonchi) can be heard when there are loud, low-pitched, rumbling, and coarse sounds during inspiration. These sounds can also be clearly heard while the client is coughing. Rhonchi may be caused by muscular spasms in the larger airways. Inflammation of the pleura may lead to a pleural friction rub sound. A crackling sound can be heard when there is a reinflation of groups of alveoli. In case of high-velocity airflow through an obstructed airway, wheezes or sibilant wheeze sounds may be heard.

The nurse is aware that the nursing diagnosis should follow the North American Nursing Diagnosis Association International (NANDA-I) label. How should the nurse document the nursing diagnosis in a three-part format?

NANDA-I label, related factor, and defining characteristics The three-part nursing diagnosis label consists of the NANDA-I label, related factor, and defining characteristics. This format is also known as the problem, etiology, and symptoms (PES) format. The nurse does not document the nursing diagnosis as NANDA-I label, related factor, and etiologies. A related factor is a condition or etiology that gives a context for the defining characteristics. The nurse does not document the nursing diagnosis as NANDA-I label, risk factor, and nursing interventions. A risk for nursing diagnosis uses the risk factor instead of related factor. Nursing interventions are not included in a nursing diagnosis. Therefore, the nurse does not document the nursing diagnosis as NANDA-I label, related factor, nursing interventions.

A client complains of rapid, involuntary movement of the eyes after a minor eye injury. A nurse assesses the client and finds that it is a disorder of the cranial nerves. Which condition does the nurse suspect?

Nystagmus Nystagmus is a condition defined by rapid, involuntary, rhythmical oscillation of the eyes. This condition is caused by local injury to the eye muscles and supporting structures. A cataract is a condition in which the opacity of the lens will be increased; this disorder is commonly related to age. Glaucoma is intraocular structural damage resulting from elevated intraocular pressure. Strabismus is a congenital condition in which both eyes do not focus on an object simultaneously. In this condition, the eyes appear crossed.

While auscultating the heart, a healthcare provider notices S 3 heart sounds in four clients. Which client is at more risk for heart failure?

Older adult client The S3 is the third heart sound heard after the normal "lub-dub." It is indicative of congestive heart failure in adults over 30 years old. In young, pregnant, and under 30 year old clients, the third heart sound is often considered to be a normal parameter.

A registered nurse (RN) must assess the body temperature of a client with a history of epilepsy. Which site for measuring temperature is contraindicated in this client?

Oral cavity The oral cavity is not a preferred site to measure the body temperature of a client with epilepsy, oral surgery, trauma, or shaking chills. Epileptic clients become rigid during seizures and any sudden seizure attack during temperature measurement poses the risk of breaking the thermometer in the mouth, lacerations, accidental mercury ingestion, and possibly aspirating the broken pieces. The skin, axilla, and temporal artery are sites that can be safely used to measure topical body temperature in an epileptic client.

A registered nurse (RN) is instructed to assess the body temperature of a neonate. Which site for placing the thermometer is contraindicated in these clients?

Oral cavity The oral cavity is the preferred site for temperature measurement in adult clients. This site is contraindicated for neonates and unconscious or uncooperative clients. The axilla is a safe site for placing a thermometer in neonates. The temporal artery is indicated for rapid temperature measurement. This site is indicated for premature infants, newborns, and children. The tympanic membrane is indicated in newborns to reduce infant handling and heat loss.

Which client assessment finding should the nurse document as subjective data?

Pain rating of 5 Subjective data are obtained directly from a client. Subjective data are often recorded as direct quotations that reflect the client's feelings about a situation. Vital signs, laboratory results, and pulse oximetry are examples of objective data.

Which nursing action can be inferred from the following figure? Pic of hand above female's clavicle

Palpation of the supraclavicular lymph node The nurse is assessing a client by palpating the supraclavicular lymph nodes, which are found just above the clavicle. The submental lymph node is present just below the chin. The submandibular lymph node is located at the underside of the jaw on either side. The external jugular lymph node is beneath the sternocleidomastoid muscles.

The nurse discovers several palpable elevated masses on a client's arms. Which term most accurately describes the assessment findings?

Papules Papules are superficial and elevated up to 0.5 cm. Nodules and tumors are masses similar to papules, but are elevated more than 0.5 cm and may infiltrate deeper into tissues. Erosions are characterized as loss of the epidermis layer; macules are nonpalpable, flat changes in skin color less than 1 cm in diameter; and vesicles are usually transparent, filled with serous fluid, and are a blisterlike elevation.

A nurse assesses for hypocalcemia in a postoperative client. What is one of the initial signs that might be present?

Paresthesias (abnormal dermal sensation (e.g., a tingling, pricking, chilling, burning, or numb sensation on the skin) with no apparent physical cause.) Normally, calcium ions block the movement of sodium into cells. When calcium is low, this allows sodium to move freely into cells, creating increased excitability of the nervous system. Initial symptoms are paresthesias. This can lead to tetany if untreated. Headache, pallor, and blurred vision are not signs of hypocalcemia.

What is the inflammation of the skin at the base of the nail called?

Paronychia Paronychia is the inflammation of skin at the base of nail. Concavely curved nails are called koilonychias. Transverse depressions in nails indicating a temporary disturbance of nail growth are called Beau lines. Red or brown linear streaks in the nail bed are caused by minor trauma to nails, subacute endocarditis, or trichinosis and are called splinter hemorrhages.

The nurse has just arrived to begin a shift in a healthcare facility nursing unit. Two new clients have been recently admitted to the unit. What should the nurse do first to collect the initial information about the assigned clients?

Participate in shift report The nurse should participate in shift report with the healthcare team from the previous shift. The nurse who is completing care for one shift prepares the change-of-shift report to communicate client details to the nurse for the next shift. These end-of-shift bedside rounds provide patient-centered care as the current shift nurse shares information about the client's condition, status of problems, and treatment plan with the nurse of the oncoming shift. Nurses usually review the client's medical reports and meet the client after completing shift report. The nurse also can meet the client's family after obtaining firsthand information from the nurse completing the shift.

A client with internal bleeding is in the intensive care unit (ICU) for observation. At the change of shift an alarm sounds, indicating a decrease in blood pressure. What is the initial nursing action?

Perform an assessment of the client before resuming the change-of-shift report The cause of the alarm should be investigated and appropriate intervention instituted; after the client's needs are met, then other tasks can be performed. An alarm should never be ignored; the client's status takes priority over the change-of-shift report. The diastolic pressure limit has been prescribed by the primary healthcare provider and should not be changed for the convenience of the nurse. Alarms always should remain on; the alarm indicates that the client's blood pressure has decreased and immediate assessment is required.

Which step of the nursing process is directly affected if the nurse does not make a nursing diagnosis?

Planning The planning phase of the nursing process is directly affected if the nurse does not make a nursing diagnosis. The nurse cannot plan or interpret correctly if the client's problems are not clear. The evaluation phase of the nursing process is not directly affected by the nursing diagnosis. A nursing diagnosis is based on an accurate assessment. The nurse must obtain and document a comprehensive assessment. In the absence of nursing diagnosis, the nurse cannot implement appropriate nursing interventions. The implementation phase is directly affected if there is no plan of care.

A client has relocated to a new city for work. The client is unable to continue the practice of walking for 30 minutes daily and exercising five days a week. Which stage of the transtheoretical model of health behavior change is the client experiencing?

Precontemplation The client is experiencing a relapse while attempting to make behavioral changes to his or her lifestyle. When relapse occurs, the client returns to the contemplation or precontemplation stage before attempting to change again. The action stage lasts for up to six months during which the client is actively engaged in strategies to change behavior. During the preparation stage, the client begins to believe that advantages outweigh disadvantages of behavior change. The maintenance stage begins six months after the change has started and continues indefinitely.

An 82-year-old retired schoolteacher is admitted to a nursing home. During the physical assessment, the nurse may identify which ocular problem common to persons at this client's developmental level?:

Presbyopia Presbyopia is the decreased accommodative ability of the lens that occurs with aging. Tropia (eye turn) generally occurs at birth. Myopia (nearsightedness) can occur during any developmental level or be congenital. Hyperopia (farsightedness) can occur during any developmental level or be congenital.

What does a nurse consider the most significant influence on many clients' perception of pain when interpreting findings from a pain assessment?

Previous experience and cultural values Interpretation of pain sensations is highly individual and is based on past experiences, which include cultural values. Age and sex affect pain perception only indirectly because they generally account for past experience to some degree. Overall physical condition may affect the ability to cope with stress; however, unless the nervous system is involved, it will not greatly affect perception. Intelligence is a factor in understanding pain so it can be tolerated better, but it does not affect the perception of intensity; economic status has no effect on pain perception.

The nurse at a community healthcare center focuses on providing primary preventive care. What is the focus of primary preventive care?

Promoting health in healthy individuals Primary Prevention - trying to prevent yourself from getting a disease. Secondary Prevention - trying to detect a disease early and prevent it from getting worse. Tertiary Prevention - trying to improve your quality of life and reduce the symptoms of a disease you already have. Primary prevention precedes disease or dysfunction and is applied to clients considered physically and emotionally healthy. Health education programs, immunizations, and physical and nutritional fitness activities are primary prevention activities. Tertiary preventive care occurs when an individual has a permanent or irreversible disability. The client undergoing rehabilitation is receiving tertiary preventive care. Secondary preventive care focuses on individuals who are experiencing health problems. Secondary preventive care involves treating clients in the early stages of disease. It also focuses on preventing complications from illness.

Which position is indicated to assess the musculoskeletal system and is contraindicated in clients with respiratory difficulties?

Prone position Prone position is indicated to assess the musculoskeletal system in clients, but it is indicated with caution in clients with respiratory difficulties because they cannot tolerate this position well. Sims position is indicated to assess the rectum and vagina. Supine position is indicated for general examination of head and neck, anterior thorax, breast, axilla, and pulses. Knee-chest position is indicated for rectal assessment.

A nurse must establish and maintain an airway in a client who has experienced a near-drowning in the ocean. For which potential danger should the nurse assess the client?

Pulmonary edema Additional fluid from surrounding tissues will be drawn into the lung because of the high osmotic pressure exerted by the salt content of the aspirated ocean water; this results in pulmonary edema. Hypoxia and acidosis may occur after a near-drowning, not alkalosis. Renal failure is not a sequela of near-drowning. Hypovolemia occurs because fluid is drawn into the lungs by the hypertonic saltwater.

While caring for a postoperative client, the nurse observed a pulse deficit during physical assessment. Which pulses are used to assess the pulse deficit?

Radial and apical pulse Pulse deficit may be associated with an abnormal rhythm. Pulse deficit is the difference between the radial and apical pulse. The carotid pulse is measured when a client's condition worsens suddenly. The brachial pulse is used to measure blood pressure. The temporal pulse is used to assess the pulse in children.

The nurse documents the data gathered during the assessment in a client's medical record. What should the nurse do to ensure that the data is meaningful to other healthcare providers?

Record objective information using accurate terminology The nurse should document all objective information using accurate terminology. The nurse should pay attention to the facts and report findings exactly as seen, felt, or smelled. If the information is not specific, another healthcare provider reading the data gets only general impressions. The nurse should record subjective information in quotations, exactly as described by the client. The nurse should refrain from generalizing or forming judgments during documentation. This information is used to form nursing diagnoses, which must be factual and accurate. During validation, the nurse compares data from the physical examination with client behavior.

While assessing a client with chills and fever, the nurse observes that the febrile episodes are followed by normal temperatures and that the episodes are longer than 24 hours. Which fever pattern does the nurse anticipate?

Relapsing Periods of febrile episodes coupled with periods of acceptable temperature values is a relapsing type of fever. These periods are often longer than 24 hours. In a sustained fever, the body temperature remains constantly above 38 oC with little fluctuations. In a remittent fever, the fever spikes and falls without a return to normal temperature levels. If the temperature returns to an acceptable value at least once in 24 hours, the fever is termed intermittent.

A client is admitted with metabolic acidosis. The nurse considers that two body systems interact with the bicarbonate buffer system to preserve healthy body fluid pH. What two body systems should the nurse assess for compensatory changes?

Respiratory and urinary Increased respirations blow off carbon dioxide (CO 2), which decreases the hydrogen ion concentration and the pH increases (less acidity). Decreased respirations result in CO 2 buildup, which increases hydrogen ion concentration and the pH falls (more acidity). The kidneys either conserve or excrete bicarbonate and hydrogen ions, which helps to adjust the body's pH. The buffering capacity of the renal system is greater than that of the pulmonary system, but the pulmonary system is quicker to respond. Skeletal and nervous systems do not maintain the pH, nor do muscular and endocrine systems. Although the circulatory system carries fluids and electrolytes to the kidneys, it does not interact with the urinary system to regulate plasma pH.

The nurse is assessing a client after surgery. Which assessment finding does the nurse obtain from the primary source?

Severity of pain primary source of information during an assessment: the client. The nurse gathers information about the client's pain from the primary source, the client. Medical records such as x-ray reports and results of blood work are secondary sources of information. The client's family caregiver is a secondary source of information.

A client suspected to have a prostate disorder is encouraged to have a rectal examination. What position of the client will facilitate a rectal examination by the registered nurse (RN)?

Sims position In Sims position, hips and knees are flexed, which results in exposure of the rectal area. Therefore Sims position is most suitable for performing rectal examinations. A prone position helps in assessing extension of hips, skin, and buttocks. The dorsal recumbent position is predominantly indicated for abdominal assessment because it promotes abdominal muscle relaxation. The lateral recumbent position is indicated for detecting heart murmurs.

The nurse pulls up on the client's skin and releases it to determine whether the skin returns immediately to its original position. What is the nurse assessing for?

Skin turgor Skin turgor is assessed by gently pinching the skin and releasing it while observing the degree of elasticity. If the skin pinch remains elevated or is slow to return to its original position, this may be an indication of dehydration or deficient fluid volume. This assessment technique is not appropriate for assessing pain tolerance, checking for ecchymosis formation, or measuring tissue mass.

When teaching about aging, the nurse explains that older adults usually have what characteristic?

Slower reaction times A decrease in neuromuscular function slows reaction time. The ability to be flexible has less to do with age than with character. Confusion is not necessarily a process of aging, but it occurs for various reasons such as multiple stresses, perceptual changes, or medication side effects. Most older adults do not have organic mental disease.

When nurses are conducting health assessment interviews with older clients, what step should be included?

Spend time in several short sessions to elicit more complete information from the clients Spending time in several short sessions reduces client fatigue and compensates for a shortened attention span, which is common in the older adult. The questionnaire may never be completed if it is left for the client to complete at their leisure. Asking family members rather than the client to supply the necessary information is degrading to the client; the client should be asked initially and, if necessary, family can be asked to fill in details later. Constantly referring to previous questions may be overwhelming and create feelings of anger and resentment.

While performing a physical assessment in a client, the registered nurse (RN) notices reddish linear streaks in the nail bed. Which systemic condition can the registered nurse (RN) suspect in the client based on these assessment findings?

Subacute bacterial endocarditis Red or brown linear streaks in the nail bed are caused by minor trauma to nails, subacute bacterial endocarditis, or trichinosis and are called splinter hemorrhages. Conditions such as syphilis and iron deficiency anemia cause concavely curved nails, called koilonychia. Heart and lung abnormalities such as chronic obstructive pulmonary disease cause clubbing of the nail beds.

Which assessment should the nurse exclude when dealing with a client with receptive and expressive aphasia?

Test mental status by asking for feedback from the client Receptive and expressive aphasia are the two types of aphasia. A client with receptive is unable to understand written or verbal speech. A client with expressive aphasia understands written and verbal speech but cannot write or speak appropriately. A client with aphasia may not have the mental ability to give feedback; asking for feedback is ineffective. Asking the client to read simple sentences aloud is an effective way of dealing with this client. Pointing to a familiar object and asking the client to name it is also effective. A client with aphasia can understand simple verbal commands.

A registered nurse is supervising a student nurse while assessing a 70-year-old client who is receiving aminoglycoside therapy. Which statement about the client's condition requires correction?

The client may have thinning of the tympanic membrane An older adult who is on aminoglycoside antibiotic therapy is at a high risk of developing ototoxicity. The client with ototoxicity may have thickening of the tympanic membrane, but not thinning of the tympanic membrane. Deterioration of the cochlea may cause older adults to gradually lose hearing. They may experience an inability to hear high-frequency sounds and differentiate between consonants.

The nurse is caring for a client who has lost an arm in a motor vehicle accident. Which reaction made by the client leads the nurse to realize that the client is in the withdrawal phase of adjusting to the change in body image?

The client recognizes the reality and becomes anxious The client with a change in body image following an injury recognizes the reality of the change, becomes anxious, and refuses to discuss it. This client uses withdrawal as an adaptive coping mechanism. During the acknowledgement phase, the client and family go through a grieving period as they acknowledge the change in physical appearance. At the end of the acknowledgement phase, they learn to accept the loss. Initially, the client is in a state of shock and depersonalizes the change. The client talks as if another person is affected by the change. The client in the rehabilitation stage is ready to learn how to adapt to the change in body image through use of prosthesis or changing lifestyles and goals.

A client experiencing chills and fever is admitted to the hospital. After assessing the client's vitals and medical history, the nurse concluded that the client's fever pattern is remittent. Which assessment finding led to this conclusion?

The client's fever spikes and falls without a return to normal temperature levels In a remittent pattern of fever, the fever spikes and falls without a return to normal temperature levels. If the temperature returns to an acceptable value at least once in a 24 hour interval, the fever has an intermittent pattern. Periods of febrile episodes and periods with acceptable temperature values is a relapsing type of fever. In a sustained fever, the body temperature is constantly above 38°C and has little fluctuation.

The nurse is developing a nursing diagnosis for a client after surgery. The nurse documents the "related to" factor as first time surgery. Which assessment activity enabled the nurse to derive this conclusion?

The nurse asks the client to explain the surgery The nurse must assess the client's knowledge about the surgery to determine if the client is aware of the outcome of surgery. The nurse observes for nonverbal signs of discomfort because some clients may not state that they are in pain. The nurse observes the client's positioning in bed to determine any abnormal signs such as discomfort or pain. The nurse asks the client to rate the severity of pain to determine a nursing diagnosis of pain related to a surgical wound.

Following assessment, a nurse documents auscultation of course rhonchi in the anterior upper lung fields bilaterally that clears with coughing. What would be the cause of these sounds?

Turbulence due to muscular spasm and fluid or mucus in the larger airways Loud, low pitched, rumbling coarse sounds heard over the trachea and bronchi are due to turbulence caused by muscular spasm when fluid or mucous is present in the larger airways. Pleural rub produces a dry or grating quality sound, best heard in the lower portion of the anterior lateral lung. Random and sudden reinflation of groups of alveoli produces crackling sounds predominantly heard in the left and right lung bases. High-velocity airflow through severely narrowed or obstructed airways results in a wheezing sound heard all over the lung.

Which pulse site is used for the Allen's test?

Ulnar The ulnar site is used for the Allen's test. The popliteal pulse is used to assess status of circulation to lower leg. The status of the circulation in the lower arm and blood pressure are assessed using the brachial pulse. The femoral pulse is used to assess the character of the pulse during physiological shock or cardiac arrest when other pulses are not palpable.

The nurse is caring for a client whose forehead feels warm to the touch. The nurse uses a thermometer and obtains the client's temperature. What is the nurse doing?

Validation The nurse is validating the presence of fever in the client. Validation is the process of gathering more assessment data. It involves clarifying vague or unclear data. Assessment is the first step of the nursing process. It involves collecting information from the client and secondary sources. During interpretation, the nurse recognizes that further observations are needed to clarify information. Data documentation is the last part of a complete assessment. The nurse must document facts in a timely, thorough, and accurate manner to prevent information from getting lost.

A client with osteoporosis is encouraged to drink milk. The client refuses the milk, explaining that it causes gas and bloating. Which food should the nurse suggest that is rich in calcium and digested easily by clients who do not tolerate milk?

Yogurt Yogurt, which contains calcium, is digested more easily than milk because it contains the enzyme lactase, which breaks down milk sugar. Yogurt contains approximately 274 to 415 mg of calcium for an 8 oz (237 mL) container depending on how it is prepared. Oats have approximately less than 50 mg of calcium/serving (depends on the brand). One potato contains approximately 7 to 20 mg of calcium depending on how it is prepared. Eight ounces of applesauce contain approximately 3 mg of calcium.

While assessing an older adult during a regular health checkup, a nurse finds signs of elder abuse. Which physical findings would further confirm the nurse's suspicion? Select all that apply. Presence of... hyoid bone damage cognitive impairment burns from cigarettes bed sores unexplained bruises on the wrist(s)

burns from cigarettes bed sores unexplained bruises on the wrists A physical finding of abuse in older adults can be the presence of burns from cigarettes. The physical presence of bed sores also indicates client abuse. Unexplained bruises on the wrist(s) may also be an indication of abuse in older adults. The presence of hyoid bone damage is an indication of intimate partner violence. The presence of cognitive impairment is a behavioral finding in older adult abuse.

The registered nurse is teaching a nursing student about bulimia nervosa in adolescents. Which statement made by the nursing student indicates effective learning?

client experiences recurrent episodes of binge eating Bulimia nervosa is an eating disorder in which the client has an obsessive desire to lose weight. In this condition, bouts of extreme overeating are followed by fasting or self-induced vomiting. A recurrent episode of binge eating is an indicator of bulimia nervosa. A client claims to feel fat despite being underweight may have anorexia nervosa. Other assessment findings of anorexia nervosa include an intense fear of gaining weight despite being underweight and a refusal to maintain a body weight over a minimal ideal body weight.

The nurse is gathering a client's health history. Which information does should the nurse classify as biographical information? Select all that apply. Symptoms client's age family structure type of insurance occupational status

client's age type of insurance occupational status Biographical information is factual demographic data about the client usually obtained by the admitting office staff. The client's age, types of insurance, and occupation status are considered biographical information. If the client presents with an illness, the nurse gathers details about the symptoms of the illness, which is descriptive information, not biographical information. The nurse obtains information about family structure while assessing the family history of the client. It is not biographical information.

Which client is suspected to have an increased risk of hyperlipidemia? Select all that apply. client with... corneal arcus periorbital edema decreased skin turgor paleness of conjunctivae yellow lipid lesions on eyelids

corneal arcus yellow lipid lesions on eyelids The presence of corneal arcus, which is the whitish opaque ring around the junction of the cornea and sclera, indicates that the client has hyperlipidemia. Yellow lipid lesions on the eyelids refer to xanthelasma, which indicates a client has hyperlipidemia. The presence of periorbital edema indicates the client may have kidney disease. Decreased skin turgor may be due to dehydration. Paleness of the conjunctivae indicates anemia.

While assessing the client's skin, a nurse notices a skin condition, the pathophysiology of which involves increased visibility of oxyhemoglobin caused by an increased blood flow due to capillary dilation. Which condition is associated with this client?

erythema Erythema occurs due to an increased visibility of oxyhemoglobin, which is caused by increased blood flow. Pallor is caused by a reduced amount of oxyhemoglobin or a reduced visibility of oxyhemoglobin. Vitiligo is a pigmentation disorder caused by autoimmune diseases. Cyanosis is a bluish discoloration of the skin around the lips; this occurs due to an increased amount of deoxygenated hemoglobin in the blood.

What should the nurse do when the defining characteristics of assessment data for a client can apply to more than one diagnosis? Select all that apply. reassess client reject all diagnoses gather more information identify related factors review all defining characteristics

gather more information identify related factors review all defining characteristics The nurse must gather more information to clarify interpretations of assessment data. Correct interpretation of information allows the nurse to select the right diagnosis that applies to the client. A related factor is a condition or etiology that gives a context for the defining characteristics. The nurse should identify related factors to individualize a nursing diagnosis for the client. The nurse should review all the defining characteristics, eliminate irrelevant ones, and confirm the relevant ones. The nurse must interpret the data to form data clusters only after reassessing and validating it. At this stage, the nurse should have only validated assessment data in the database. The nurse need not reject all diagnoses. The nurse should review all the defining characteristics to support or eliminate the irrelevant ones.

A nurse is performing an eye assessment in an older adult. The older adult is unable to see near objects. Which conditions may be suspected in the older adult? Select all that apply. cataract glaucoma hyperopia presbyopia macular degeneration

hyperopia presbyopia In hyperopia, the client has farsightedness. In this condition, the client is unable to see near objects. Presbyopia is an impaired near vision that may occur with aging. Therefore, the nurse can suspect either of the conditions. In cataracts, there is an increased opacity of the lens that blocks light rays from entering the eye, leading to impaired vision. Glaucoma is a condition in which there is intraocular structural damage resulting from elevated intraocular pressure. Macular degeneration is caused due to blurred central vision that often occurs suddenly. This is caused by a progressive degeneration of the center of the retina.

How does the nurse identify an illness as chronic? Select all that apply. illness is reversible and often severe illness persists for longer than 6 months client may develop life threatening relapse symptoms are intense and appear abruptly illness affects the functioning of 1 or more systems

illness persists for longer than 6 months client may develop life threatening relapse illness affects the functioning of 1 or more systems A chronic illness usually lasts longer than six months. The client with chronic illness often fluctuates between maximal functioning and serious health relapses that may be life threatening. The illness affects the functioning of one or more systems. A chronic illness is irreversible, whereas an acute illness is reversible and often much more severe than a chronic illness. The client with acute illness develops intense symptoms that appear abruptly and often subside after a relatively short period.

Which body temperature measurement sites would be considered safe, inexpensive, and least invasive? Select all that apply. skin oral axilla rectal tympanic

skin axilla The skin and axilla are the body temperature measurement sites that are considered safe, inexpensive, and least invasive. The oral route is an easily accessible site for temperature measurement, but it is invasive and increases the nurse's risk for body fluid exposure. The rectal route is also considered invasive, it is often not easily accessible, and it poses an increased risk of body fluid exposure. The tympanic route is an easily accessible site for temperature measurement, but it is invasive, and care should be taken when used in neonates, infants, and children.

Which questions should the nurse ask the client when obtaining the health history? Select all that apply. tell me about your food habits? do you use alcohol or tobacco? have you sustained any personal loss recently? have you ever experienced any allergic reactions? does any family member have a long term illness?

tell me about your food habits? do you use alcohol or tobacco? have you ever experienced any allergic reactions? The health history of a client includes the client's food habits so that the nurse can obtain an assessment of the client's nutrition status. The nurse also assesses the client's habits and lifestyle patterns. The use of alcohol and tobacco helps to determine the client's risk for diseases involving the liver or lungs. The health history includes descriptions of allergies and reactions to food, latex, drugs, or contact agents such as soap. While assessing the family history, the nurse assesses the client for stress-related problems by asking about recent personal losses. The family history provides information about family members to determine the risk for illnesses of a genetic or familial nature.


Set pelajaran terkait

General Mathematics - Number Systems and Sets

View Set

NovSpA - DLU1 - Tpc02A - Cuando nací: Los 12 meses del año

View Set

Vocabulary Unit 4 Argumentative and Persuasive Text

View Set

Loops(for loop, while loop, do while loop) - Basic Java Tutorials For Selenium WebDriver

View Set

Module 5: Cancer and the Immune System

View Set

PN Comprehensive Online Practice 2020 A

View Set